LSAT and Law School Admissions Forum

Get expert LSAT preparation and law school admissions advice from PowerScore Test Preparation.

 Administrator
PowerScore Staff
  • PowerScore Staff
  • Posts: 8917
  • Joined: Feb 02, 2011
|
#22654
Question #22: Weaken—Except, CE. The correct answer choice is (A).

Since the key to weakening an LSAT argument is to focus on the conclusion, it is essential to break down the argument, which is structured as follows:
Premise—From 1996 to 2004, the average family income decreased by 10%.

Conclusion—The ruling party during that time mismanaged the economy.
The opponents’ argument is causal, making it vulnerable to attack:

..... Cause ..... ..... Effect

..... Ruling party :arrow: Average family income decreased

To weaken this argument, you can show that an alternative cause led to the observed effect. Or, you can show that the data is potentially misleading. Because there are dozens of alternative causes that can potentially weaken the opponents' position, test makers are asking a Weaken-EXCEPT question: four of the five “rejoinders” (or replies) would directly counter the opponents’ explanation of the decrease in average family income. Whichever answer choice does NOT counter their explanation will be correct.

Answer choice (A): This is the correct answer choice. The fact that family income rose in 1996 is consistent with the observation that it decreased from 1996 until 2004. This answer choice has no bearing on the conclusion of the argument, making it the correct answer choice to this Weaken-EXCEPT question.

Answer choice (B): This answer choice suggests that average family income may have decreased for noneconomic reasons. If so, this would be an alternative cause for the observed effect, weakening the opponents’ argument.

Answer choice (C): Like answer choice (B), this one suggests an alternative cause for the observed effect—international events beyond the control of the country’s government had an adverse effect on family incomes. This weakens the opponents’ argument, and is therefore incorrect.

Answer choice (D): If the average age of household wage earners fell during that time, and younger people tend to earn less, it would be reasonable to expect that the average family income would decrease. Assuming that the ruling party cannot be blamed for the shifting demographics, this answer choice weakens the opponents’ accusations and is therefore incorrect.

Answer choice (E): If policies enacted before the ruling party came to power resulted in the biggest decreases in family income, then the ruling party is not (entirely) to blame for the decreased income. As with answer choices (B) and (C), we have an alternative explanation for the decreased income, weakening the opponents’ position.
 mpoulson
  • Posts: 148
  • Joined: Mar 25, 2016
|
#23316
Hello,

Can you let me know if I am on the right track for answer A? From my perspective, if the rise in family income happened in 1996 it could still decrease over the next ten years and wouldn't effect the opponents conclusion. Thank you.

Respectfully,

Micah
 David Boyle
PowerScore Staff
  • PowerScore Staff
  • Posts: 836
  • Joined: Jun 07, 2013
|
#23358
mpoulson wrote:Hello,

Can you let me know if I am on the right track for answer A? From my perspective, if the rise in family income happened in 1996 it could still decrease over the next ten years and wouldn't effect the opponents conclusion. Thank you.

Respectfully,

Micah

Hello,

That seems right, that the average income could still decline up to 2004, even if there were one good year for family income, 1996.

David
 HowardQ
  • Posts: 32
  • Joined: Jun 25, 2018
|
#49450
Hi,

I'm a little confused by the wording "From 1996 to 2004" Does that mean someday in 1996 to someday in 2004? Also, the political party ruled during this time could mean either the entire time interval or a portion of the interval. In this case, these assumptions could affect the validity of A and E. Is there a general rule when these wording appear?

Thanks,
 Adam Tyson
PowerScore Staff
  • PowerScore Staff
  • Posts: 5153
  • Joined: Apr 14, 2011
|
#49725
I would interpret that as meaning beginning in 1996 and ending in 2004, and probably including most of 1996 and perhaps some, but not much, of 2004, HowardQ. If it was from late 1996 until early 2004 they probably would have said "between" those years rather than "from" one year to the other. If it included all of 2004 they would probably have said "through 2004" rather than "to 2004." If I say I work from 9 to 5, I mean I start at 9 and end at 5. It's imprecise, but that should not affect your analysis much, if at all. The issue is not what happened at any one moment in that time span, but the trend across that entire span. Whatever happened at the beginning of 1996 or the end of 2004 is of less importance than the overall trend and what caused it.

I would also interpret the language about the ruling party meaning they ruled the entire time. Otherwise, the claim would make no sense. That would be like saying "the political party that ruled the United States from 1980 through 2000" when in fact two different parties swapped rule during that time. Perhaps focus on the difference between "the" party and "a" party - "the" means there was just one, while "a" indicates there was more than one.
User avatar
 rragepack
  • Posts: 22
  • Joined: Jan 22, 2021
|
#91712
If B were not correct, wouldn't we making the assumption that "multiple incomes=high income"?

Thank you.
 Adam Tyson
PowerScore Staff
  • PowerScore Staff
  • Posts: 5153
  • Joined: Apr 14, 2011
|
#91744
B weakens the argument, rragepack, because it indicates that the reduction in total family income may be due to noneconomic reasons, and therefore not due to the ruling party mismanaging the economy. It's not so much that we must assume that fewer incomes in a given family means a lower total income, but just that fewer incomes could help to explain a lower total family income. This answer is saying "there is another factor that should be considered," thus weakening the argument even if the answer doesn't prove anything.
User avatar
 Henry Z
  • Posts: 61
  • Joined: Apr 16, 2022
|
#95302
Why is (E) wrong?

I chose (E) for two reasons:
1) Should we assume "policies enacted before the ruling party came to power in 1996" simply means policies in 1995? What if it's 1970? Or even 1900?
2) Even if it's due to policies enacted in 1995, why it's not a "mismanagement" when the ruling party fails to save the economy?
User avatar
 katehos
PowerScore Staff
  • PowerScore Staff
  • Posts: 184
  • Joined: Mar 31, 2022
|
#95380
Hi Henry!

When we look at answer choice (E), we can see that if the ruling party didn't come to power until 1996, then policies enacted at any point prior to 1996 were not enacted by the ruling party from 1996-2004! So, to answer your first point, it doesn't matter if the policies were enacted in 1900 or 1970, since no matter what, those policies weren't enacted by the party in question.

To your second point, we could certainly say that policies prior to 1996 may be 'mismanagement' of the economy, but not by the ruling party the opponents are referring to. So, answer choice (E) does weaken the opponents' claim by shifting the blame from the party from 1996-2004 to another party entirely!

I hope this helps! :)
-Kate
User avatar
 Henry Z
  • Posts: 61
  • Joined: Apr 16, 2022
|
#95392
katehos wrote: Tue May 17, 2022 12:04 pm Hi Henry!

When we look at answer choice (E), we can see that if the ruling party didn't come to power until 1996, then policies enacted at any point prior to 1996 were not enacted by the ruling party from 1996-2004! So, to answer your first point, it doesn't matter if the policies were enacted in 1900 or 1970, since no matter what, those policies weren't enacted by the party in question.

To your second point, we could certainly say that policies prior to 1996 may be 'mismanagement' of the economy, but not by the ruling party the opponents are referring to. So, answer choice (E) does weaken the opponents' claim by shifting the blame from the party from 1996-2004 to another party entirely!

I hope this helps! :)
-Kate
Thanks for your reply, Kate. I can see why (A) is correct. But I'm still confused about how (E) "directly counters" the opponents' explanation. To me, it's like when someone today criticizes the White House for their economic mismanagement, the White House replies:"The biggest economic mismanagement was during the Great Depression!" That's a bad counter-argument.

To clarify, my first point is meant to point out that (E) only says "before" 1996, but doesn't specify how long ago. If the policies and the consequent decreases were a long time and multiple administrations ago, say in 1900, then those things are minimally irrelevant by 1995. It could well be a booming economy when the party took over in 1996. So I don't think the point is whether the ruling party themselves enacted the bad policies, but whether those policies still have consequences when the party came to power. To make (E) a weakener (and thus a wrong answer), we have to assume that those policies are recent enough, like in 1995. I don't see how we're allowed to make such an assumption.

My second point is that just because another party mismanaged, it doesn't mean the ruling party from 1994 to 2004 didn't mismanage. I can't see how by pointing out there was a mismanagement prior to 1996, (E) just shifts "the blame from the party from 1996-2004 to another party entirely!" Isn't this kind of whataboutism a classic flaw in LSAT? Why should we see it as a legit weakener here?

Get the most out of your LSAT Prep Plus subscription.

Analyze and track your performance with our Testing and Analytics Package.